How Do You Prove a General Solution for Non-Homogeneous Differential Equations?

Click For Summary
To prove the general solution for non-homogeneous differential equations, consider the difference F(t) - V(t), where F(t) is the general solution and V(t) is a particular solution. The discussion emphasizes that F(t) can be expressed as F(t) = U(t) + V(t), with U(t) representing the general solution of the associated homogeneous system. It is suggested to label the equations clearly and verify if F(t) - V(t) satisfies the original non-homogeneous equation. This approach aligns with the standard theorem for solving non-homogeneous differential equations. The conversation encourages checking the validity of the derived solutions through substitution.
lalaman
Messages
1
Reaction score
0
Hello all, I am currently having trouble with this Differential Equations problem.

Let x = F(t) be the general solution of x'=P(t)x+g(t), and let x=V(t) be some particular solution of the same system. By considering the difference F(t)−V(t), show that F(t)=U(t)+V(t), where U(t) is the general solution of the homogeneous system x'=P(t)x.

Attempt:

Since F is a solution, we know that there exists a fundamental matrix such that M such that F=MW, where W is such that MW′=g. But that is all I have been able to deduce. Also, I am not sure if F(t) - V(t) would be considered a soluton as well.

Thank you for your time. :)
 
Last edited:
Physics news on Phys.org
Welcome to PF;
It looks like you are being asked to prove the usual theorem that is used to solve non-homogeneous DEs - generalized for a system of DEs. You can always look up how it is normally done for clues.

Label your equations - (1) is the inhomogeneous equation and (2) is the associated homogeneous one.
So F is the general solution to (1) and V is a particular solution to (1).
You can easily check to see if F-V is a solution to (1) - plug it in.
 
Last edited:
Question: A clock's minute hand has length 4 and its hour hand has length 3. What is the distance between the tips at the moment when it is increasing most rapidly?(Putnam Exam Question) Answer: Making assumption that both the hands moves at constant angular velocities, the answer is ## \sqrt{7} .## But don't you think this assumption is somewhat doubtful and wrong?

Similar threads

  • · Replies 3 ·
Replies
3
Views
2K
Replies
3
Views
2K
  • · Replies 7 ·
Replies
7
Views
2K
  • · Replies 1 ·
Replies
1
Views
2K
Replies
2
Views
1K
  • · Replies 1 ·
Replies
1
Views
1K
  • · Replies 5 ·
Replies
5
Views
2K
Replies
6
Views
2K
Replies
4
Views
2K
  • · Replies 5 ·
Replies
5
Views
2K